PT16.S2.Q14 - in 1980, country a

skrishnanskrishnan Alum Member
edited November 2017 in Logical Reasoning 209 karma

I'm a bit confused with the wording in the right answer choice-
This is how I worked on this question, please let me know if there is a different way for getting this question right -

1980 (A) GDP = 10,000
1980 ( EEC) GDP = 5000

1990 (A) GDP =11,000
1990 (EEC) GDP = 5000
OR
1990 (A) GDP= 10,000
1990 (EEC) GDP= 4000

I chose D through POC but still a bit unclear on the math!
Thanks in advance

Comments

  • btate87btate87 Alum Member
    782 karma

    Hi there, your math can definitely help you get to the right answer. I think the key to this one, though, is focusing on where the language itself goes astray. Here it happens in the word "difference." You definitely interpreted it correctly based on your math, but before going to the numbers, I would compare that to the conclusion: "the average standard of living...must have risen."

    My quick way of getting to the answer was to essentially treat it like a flaw question (it's worth noting they could have easily made this a flaw question using the same stimulus), and ask, "But what if both country's GDP's shrank, but the EEC's shrank more than Country A?" So I went into the answers looking for something about EEC GDP shrinking, and when I got to Answer Choice D it wasn't hard to think through the numbers to see it was correct (or that the others were way off base.

    Hope that helps!

  • BinghamtonDaveBinghamtonDave Alum Member 🍌🍌
    8689 karma

    This question is a cross section of various tricks the LSAT often uses, thank you for drawing my attention to it. I am going to try to write an explanation without getting too much into the weeds of the math here, because I believe the abstract form of this question and the ability to see what the argument is implying are the most important aspects here. The form is what counts here. What follows is an analogous argument:

    On January 1st of 2017 my bank account showed $500 more than my brother's bank account. On November 1st 2017 my bank account showed $600 more than my brother's bank account. Therefore, I must have gained money in my bank account in those months.

    Now abstractly, what have I done here? I have forced the observed phenomenon to conform to a specific hypothesis. I have forced the change in my bank account relative to my brother's account to conform to a specific hypothesis used to explain that change: that I have gained money. The facts presented are indeed consistent with that hypothesis. The core issue here that we must recognize is that the given hypothesis is not the only thing that could explain the observed phenomenon. By attributing me gaining money as the reason for the observed phenomenon, I am implicitly saying that another hypothesis is not the case.

    This is a tremendously important form to understand so I want to pause here. Lets review, this time we will use the phenomenon/hypothesis framework that might show up on weakening/strengthening questions.

    Observed phenomenon: on 1/1/2017 my bank account was $500 more than my brother's bank account
    and then
    on 11/1/2017 my bank account was $600 more than my brother's

    Hypothesis used to explain that phenomenon: I must have gained money in that time period

    What I am actually saying with that hypothesis: That on 1/1/2017 I had $1,500 in my bank account and my brother had $1,000 and on 11/1/2017 I had $1,600 in my bank account and my brother had $1,000.

    Implicitly what I am saying with the use of that hypothesis: That it is not the case that my bank account stayed the same and my brother's dropped. In the act of choosing the hypothesis I chose above, I am saying that it is not the case that I had $1,500 in my bank account on 1/1/2017 and my brother had $1,000 in his and then on 11/1/2017, I have $1,500 and my brother had $900. The preceding is a story (hypothesis) that is also consistent with the given phenomenon.

    So, what is necessary for our argument to hold? It is necessary that the alternative explanation is not the case. Because if the alternative explanation (hypothesis) is indeed the case, we cannot reasonably choose the hypothesis (explanation) we chose in our conclusion.

    This is a form that will comes up time and time again, not necessarily with numbers, but it will be something you see again. For practice with a problem like this I recommend trying to coming up with a strengthening question to help the reasoning in this argument, try coming up with a weakening question to weaken the reasoning in this argument.

    I hope this helps
    David

  • skrishnanskrishnan Alum Member
    edited November 2017 209 karma

    @btate87 Thank you so much for your explanation!
    @BinghamtonDave It is the "Lower by more than 1000" in answer choice D that confused me.

    Keeping your analogous example in mind-

    for my bank account to have increased, my brothers account should have remained the same for my hypothesis to hold true. However, if it says cannot be lowered by more than 100, I can lower my brothers bank account by 99 and that would weaken my hypothesis. I get how that makes it a NA answer choice. But I want to make sure I am able to parse through such language in a different question type. Please let me know if you've come across similar questions, I just want to make sure i've understood the concept
    Thanks in advance

Sign In or Register to comment.